LSAT and Law School Admissions Forum

Get expert LSAT preparation and law school admissions advice from PowerScore Test Preparation.

 Administrator
PowerScore Staff
  • PowerScore Staff
  • Posts: 8917
  • Joined: Feb 02, 2011
|
#37404
Complete Question Explanation

Assumption. The correct answer choice is (A)

The success of chain bookstores has led to a decrease in the number of independent bookstores.
The author concludes that book consumers have been hurt by this trend, because the variety of
books readily available to them is smaller than it would have been had the chain bookstores not
prospered. It is entirely unclear, however, how the lack of variety of readily available books has been
detrimental to book consumers.

Because this is an assumption question, the answer you select must contain a statement upon which
the argument depends, i.e. a statement that is necessary for the conclusion to be true.

Answer choice (A): This is the correct answer choice. Since the author never explicitly connected
book variety to consumer benefit, a correct Supporter Assumption needs to tie the two together.
Answer choice (A) fits the bill: consumers would be better off if the variety of readily available
books were greater than it currently is. If book variety and consumer well-being were not correlated,
the author’s conclusion would be seriously undermined.

Answer choice (B): This answer choice may seem attractive, because it lends further credibility to
the claim that chain bookstores’ success has been to the detriment of book consumers. However, just
because a statement supports the author’s conclusion does not mean it is necessary for the conclusion
to be true. Apply the Assumption Negation Technique and ask yourself, “What would the author say
to this negation?”

  • Independent bookstores typically sell the kinds of books that are available in
    chain bookstores.

The logical opposite of answer choice (B) implies that one can find the same kinds of books
in both independent and chain bookstores. Nevertheless, it is still possible that the success of
chain bookstores has been to the detriment of book consumers, either because there are far fewer
independent bookstores from which to purchase these books, or because the books are not as
readily available from chain bookstores as they would have been from the independent bookstores.
Furthermore, this answer choice does not address the main issue of whether book variety benefits
consumers.

Answer choice (C): How the average bookstore changed in size over the years has no bearing on the
issue of whether lack of variety is detrimental to consumers.

Answer choice (D): As with answer choice (C), bookstore size has no bearing on the issue of
whether book variety benefits consumers.

Answer choice (E): This answer choice should be immediately eliminated, because book prices fall
outside the scope of this argument. If we assume, however, that chain bookstores have lowered the
average price of books, and book prices are more important than selection, this might be used as an
argument in favor of chain bookstores, weakening the author’s argument.
 agroves
  • Posts: 32
  • Joined: Aug 03, 2013
|
#10905
Hello,

Can you explain why the answer to Question #9 is A instead of B?

Thank you!

Angela
 Nikki Siclunov
PowerScore Staff
  • PowerScore Staff
  • Posts: 1362
  • Joined: Aug 02, 2011
|
#10925
Angela,

Thanks for your question. Let's break the argument down:

The author observes that the success of chain bookstores has led to a decrease in the number of independent bookstores, concluding that book consumers have been hurt by this trend. Why? Because the variety of books readily available to them is smaller than it would have been had the chain bookstores not prospered. It is entirely unclear, however, how the lack of variety of readily available books has been detrimental to book consumers.

Because this is an assumption question, the answer you select must contain a statement upon which the argument depends, i.e. a statement that is necessary for the conclusion to be true.

Answer choice (A) is correct: Since the author never explicitly connected book variety to consumer benefit, a correct Supporter Assumption needs to tie the two together. Answer choice (A) fits the bill: consumers would be better off if the variety of readily available books were greater than it currently is. If book variety and consumer well-being were not correlated, the author’s conclusion would be seriously undermined. Since the logical opposite of answer choice (A) weakens the argument, answer choice (A) is an assumption upon which the argument depends.

Answer choice (B): This answer choice may seem attractive, because it lends further credibility to the claim that chain bookstores’ success has been to the detriment of book consumers. However, just because a statement supports the author’s conclusion does not mean it is necessary for the conclusion to be true. Apply the Assumption Negation Technique and ask yourself, “What would the author say to this negation?”:
  • Independent bookstores typically sell the kinds of books that are available in chain bookstores.
The logical opposite of answer choice (B) implies that one can find the same kinds of books in both independent and chain bookstores. Nevertheless, it is still possible that the success of chain bookstores has been to the detriment of book consumers, either because there are far fewer independent bookstores from which to purchase these books, or because the books are not as readily available from chain bookstores as they would have been from the independent bookstores. Furthermore, this answer choice does not address the main issue of whether book variety benefits consumers.

Hope this helps! Let me know.
 agroves
  • Posts: 32
  • Joined: Aug 03, 2013
|
#10928
Thanks the help, Nikki! This helps, although I still feel like answer choice A depends upon choice B to have relevance. However, I think I need more practice with assumption questions and am going to go back to that section in our course.

Thanks again!
 Adam Tyson
PowerScore Staff
  • PowerScore Staff
  • Posts: 5153
  • Joined: Apr 14, 2011
|
#10930
Angela, in reviewing answer choice B, consider this - does "variety" mean different "kinds" of books, or might it simply mean different books? Might independent and chain stores both sell exactly the same kinds of books (fiction, cookbooks, how-to guides, etc.), and yet independent stores still have more variety within those categories? Maybe chains carry only bestsellers in the fiction category, while independents carry new authors, local authors, etc. that aren't the top sellers. That, for me, puts the nail in the coffin for answer B. We don't need to assume they sell different kinds in order to make this conclusion - we only need to have a difference in the variety available.

Hope that helps to further explain Nikki's excellent answer!
 Bry
  • Posts: 3
  • Joined: Aug 29, 2015
|
#19568
My thought: Premise- FOR the shortage of independent bookstores has prevented the variety of readily available books from growing as much as it otherwise would have.

MP: THUS, bookstore's success has been to the detriment of book consumers.

My concern: the correct answer: If there were a greater variety of readily available books than there currently is (this looks like a negation of the premise), book consumers would be better off (this looks like a negation of the MP) = Mistaken Negation

How can this be the case? Does the correct answer have something to do with "otherwise would have"?
 Jon Denning
PowerScore Staff
  • PowerScore Staff
  • Posts: 904
  • Joined: Apr 11, 2011
|
#19613
Hey Bry,

Thanks for the question and welcome to the Forum! You're using some traditionally-PowerScore language here ("Mistaken Negation"), so I'll assume you're either in one of our courses or have read some of our books, and write this accordingly. If I've misjudged that and anything is unclear please let me know! :-D

First off, for both the sake of total clarity as well as anyone else who might stumble on this explanation and not understand the question to the extent that you have, let me walk through a general explanation of it before I address your specific concern.

This is an Assumption question, which means we can expect one of two things to happen: either there will be a missing piece of information needed to connect the conclusion to the rest of the premises (a Supporter Assumption), or there will be an idea that could severely harm the argument and the correct answer choice will rule it out (a Defender Assumption).

In this case, we have a Supporter: the conclusion is about consumer detriment/harm, which is not addressed anywhere else in the stimulus! (sidenote: I LOVE Supporters, as you can often make incredibly strong prephrases about the correct answer)

So let's consider the pieces. We have a conclusion that says chain bookstores have made people worse off (been to their detriment), because chain bookstores have reduced the number of independent bookstores, in turn limiting the variety of readily available books compared to what it others would have been.

What's the assumption? What do we need to connect? Clearly this author feels there is a link between the available variety of books and consumers' well-being. Specifically, that a reduced variety of readily available books relative to what it would have been otherwise has made consumers' situations worse than they would have been otherwise. So I'd be looking immediately for an answer choice that reflected that idea, and (A) is easily the closest of the bunch.

But you raise an interesting, and insightful, point. If we see this connection as simply "less variety :arrow: consumer harm," and (A) as "greater variety :arrow: consumer benefit," we have an apparent logical flaw on our hands in the form of a Mistaken Negation (the sufficient condition "greater variety" and the necessary condition "consumer harm" have both simply been negated, which is typically a problem).

So why isn't this one?

The answer lies in the way that its worded. The author isn't speaking in conditional absolutes so much as in causal relatives. A lesser variety than what would otherwise be has made people worse than they would otherwise be. Meaning if we reduced that cause and provided a greater variety than we have, the detrimental effect would be presumed to be less, making people better off (again, not "good" per se, simply better than at present).

So that's why (A) works! If the author thinks a reduced variety is causing consumers to be harmed, then he/she can be reasonably expected to believe (or assume) that increasing that variety would alleviate some harm, and people would therefore be better off.

Finally, when in doubt, apply the Assumption Negation technique and test (A)! If we negate it and and say "Consumers would be no better off if book variety was greater than it is now," think of how damaging that would be to the author's belief that less variety has been detrimental. (A) negated basically means variety doesn't matter, at which point the argument collapses. That's the true test of an assumption, and it works beautifully for us here! :)

I hope that helps clear things up, but please let me know if not!

Jon
 Bry
  • Posts: 3
  • Joined: Aug 29, 2015
|
#19631
Thanks for the new thought process! You were very helpful. There had been a few problems similar to this one- nonhuman/human intelligence with goal orientated behavior, international law and law enforcement passages.

I finished the class in Miami and I had a very helpful instructor.

I took the LSAT in June, I scored a 158 and now I'm churning hard to get a 165 on the October LSAT. Logical Reasoning has been hands down the most difficult section to improve on. Right now I'm stuck between 15-17 correct on each LR section and its been like that for 2months.

Something has to change, so I'm going to keep going over wrong answers from preptests 40-58 and start fresh with preptest 59 this weekend. Any tips to get out of the 15-17 correct on each LR section bubble would be much appreciated!
 jeff.wren
PowerScore Staff
  • PowerScore Staff
  • Posts: 26
  • Joined: Jul 04, 2015
|
#19638
Hi Bry,

The first thing that I'd recommend would be to look over the LR sections on any practice test that you've taken recently to see whether there are any patterns to the questions that you missed. Our score reports are a helpful tool as they show how you performed by question type, type of reasoning, etc..

For example, the question that you were asking about is an assumption question, which is often the most difficult question type for many students. It is also one of the more frequent question types in LR. If you find that you are missing a fair number of these questions (or any other question type), start by reviewing the relevant lesson. You may find watching the Virtual Modules that cover those concepts helpful.

If there are any LR questions in the homework that you haven't had a chance to finish, I'd recommend completing those. Be sure to check the online explanations for any questions that you miss.

Also, if you see that you are ever making careless errors, such as misreading a stimulus or an answer choice, this may be due to rushing and/or fatigue. If so, practice your pacing and build your mental stamina/focus in the weeks ahead by taking practice tests.

I hope this helps.

Best,
Jeff
 Bry
  • Posts: 3
  • Joined: Aug 29, 2015
|
#19688
Great advice. Thanks!

Get the most out of your LSAT Prep Plus subscription.

Analyze and track your performance with our Testing and Analytics Package.